You are on page 1of 45
WE Semiconductor Electronics: Materials, Devices and Simple Circuits _—_— Electronics isthe branch of science that deals withthe study of flow and control of electrons and their behaviour, This control ‘of electrons is accomplished by devices that resist, carry, select, steer, switch, store, manipulate, and exploit the electron Electronic instruments are being utilized in various fields like {elecommunication, entertainment, computers, nuclear physics and many more. Although the history started with the advent of vacuum tubes, however the rapid advancement in electronics which we see today is due to the valuable contributions of semiconductor devices. Semiconductor devices are small in size, consumes less power, have long life time and are of low cost. They are more efficient than vacuum tubes. That is why they have replaced vacuum tubes nearly in all applications. As an example, we can consider the case of a computer. In early days, the vacuum tube based computers were as big as the size of a room and were capable of performing simple calculations only. At present, the personal ‘computer (PC) that you seein laboratory orat your home is much smaller in size and capable of performing many operations. This became possible because of the advancements in semiconductor technology. We will lear the basic concept of semiconductors. This will enable us to understand the operation of many semiconductor devices and then we will be discussing few semiconductor devices like diode, transistor along with their applications. CLASSIFICATION OF SOLIDS ON THE BASIS OF CONDUCTIVITY (On the basis of relative values of electrical conductivity (6) or resistivity (p = 1/0), the solids are broadly classified as: Metals ‘They possess very low resistivity (or high condu p~107-10°Qm o~10'-10°S mt ty). Insulators They have very high resistivity (or low conductivity) p~ 10" 10" 2m o~ 10" 10S m" Semiconductors Their resistivity or conductivity lie between metals and insulators p~105-10°2m 9~10—10*S mr ‘The values of p and o given above are indicative of magnitudes and could also go outside the ranges as wel. |. Elemental semiconductors: Si and Ge 2. Compound semiconductors: Examples are: (i) Inorganic: CdS, GaAs, CdSe, In, etc. (ii) Organic: Anthracene, doped phthalocyanines, etc. (iii) Organic polymers: Polypyrrole, polyaniline, potythio- phene, etc. ENERGY BANDS IN SOLIDS Band Theory From Bohr’s atomic model, we know thatthe electrons have well defined energy levels in an isolated atom. But due to interatomic interactions in a crystal, the electrons of the outer shells are forced to have energies different from those in isolated atoms. Each energy level splits into several energy levels forming a continuous band, called energy band. ‘An enormously large number of energy levels closely spaced in 4 very small energy range constitute an energy band. Consider a small single crystal of silicon, Suppose it has ‘atoms, Imagine that these atoms are being brought closer from infinity 10 form a crystal of lattice spacing a (2 A or 3 A). This is depicted in figure, in which the interatomic spacing is plotted ‘long x-axis and the energy along y-axis. ~~ [| fe IN Tnteratomic spacing, r= When r=d>> a “At this lange interatomic spacing, there is no interaction between neighbouring atoms. Al the N atoms have identical energy levels. Inthe outer shells, V energy levels associated ‘with 3s orbitals are completely filled with 2V electrons Out of 3N energy levels associated with 3p orbitals, only N energy levels are filled with 2N electrons and the remaining levels are empty. When r=c>>abutea ‘Asthe separation r decreases further, the energy gap between 3s and 3p levels completely disappears and the upper and lower energy bands merge with each other. We now have a set of continuously distributed 4N energy levels. When r=a Atthis equilibrium separation, the bands of 2 filled energy levels get separated from the bands of 2N empty energy levels by an energy gap. ‘The highest energy band filled with valence electrons is called valence band. The lowest unfilled allowed energy band next to valence band is called conduction band The gap between top of valence band and bottom of the conduction band in which no allowed energy levels for clectrons can exist is called energy band gap or energy gap. Physics sre, a sold in general has two distinet band in rial may lie, The lowest conduction As shown in fi which the electrons ina mate tad energy is Band the highest vaence band energy is F, and the energy band gap ‘between them is Ei, ~ F, ~ Large number of states each occupied by thro electrons Valence Band “The energy band formed by a series of energy levels containing valence electrons is known as valence band. At OK, the electrons fill the energy levels in valence band starting from lowest one. 1, This band is always filled with electrons. 2. This is the filled band of maximum energy. 3, Electrons are not capable of gaining energy from external electric field 4, No flow of current due to electron present in this band. 5. The highest energy level which can be occupied by an clectron in valence band at 0 K is called fermi level. Conduction Band Range of energies possessed by free electrons is known as conduction band. 1. Itisalso called empty band of minimum energy 2. This band is partially filled with electrons. 3, In this band, the electron can gain energy from external electric field. 4, The electrons in the conduction band are called the free electrons. They are able to move anywhere within the volume of the solid 5, Current flows due to such electrons. Forbidden Energy Gap (AEg) It is the energy gap between conduction band and valence band. AE g = (CB)min~(VB)max ‘Semiconductor Electronics: Materials, Devices and Simple Circuits 13 CLASSIFICATION OF SOLIDS ON THE BASIS OF BAND THEORY Conductivity ofa solid ean be decided on the basis its band strueture by finding out which energy levels are actually ‘occupied and whether specific bands are empty, partially filled or completely filled. 2. Empty bands do not contain electrons and therefore do not contribute tothe electrical conductivity ofthe material 3. Partially filled bands do contain electrons as well as unoccupied energy levels which have a slightly higher ‘energy. These unoccupied energy levels enable carriers to gain energy when electric field is applied. Electrons in a partially filled band therefore do contribute to the electrical conductivity of the material. 4. Completely filled bands do contain plenty of electrons but ‘An intrinsic semiconductor at T= 0K ‘behaves like an insulator. ‘At T> OK, four thermally generated eletron-hole pars are shown, The filled circles (represent electrons and empty citles (0) represent holes. ‘Conduction band | > Hietron Hole Valence band INTRINSIC SEMICONDUCTORS ‘A semiconductor fiee from impurities is called an intinse seiiconductor.[eally, an intrinsic semiconductorcrytal sho contain atoms of this semiconductor only but i i not possible in practice to obiain crystals with such purities. However if the impurity is less than 1 in 10" part of semiconductor, i can be treated as intrinsic. For describing the properties os trinsic semiconductor, we are taking examples of silicon ang ‘germanium. Both silicon and germanium are members of the ‘group IV of periodic table of elements and are tetravalent. Their electronic configurations are as follows: Si (14)=18 25? 2p 38 3p Ge (32) = Is? 2s? ph 35° 3p! 3d" 4p? Both elements crystallize in such a way that each atom in the crystal forms a tetrahedron with the four atoms which are closest to it. Figure shows one of these tetrahedral units Each atom shares its four valence electrons with its immediate neighbours on a one to one basis, so that each atom is involved in four covalent bonds. For convenience, a two dimensional representation of the erystal structure for germanium is shown, in figure, which can also be used for silicon (as only covalent bonds are being shown). Two dimensional representation ofthe erystal structure of germanium at OK ‘ALOK, all the valence electrons are involved inthe bonding and so the crystal isa perfect insulator as there are no free electrons, available for conduction. At higher temperatures, however, some of the valence electrons have sufficient energy to break away from the bond and move in the crystal in random manner. Under an applied electric field these electrons drift and conduct electricity ‘Semiconductor Electronics: Matertals, Devices and Simple Circuits Concept of Hole and Hole Current 1 2 3 At room temperature, some of the covalent bonds in a Pure semiconductor crystal break, thus setting up fee electrons. Under the influence of electri field, these free electrons constitute electric current. At the same time, another current ie, hole current, also flows in the semiconductor. When a covalent bond is ‘broken due to thermal energy, the removal of one electron leaves a vacancy i.e. a missing electron in the covalent bond. This missing electron is called a hole which acts as a positive charge. For one electron set free, one hole is created. Therefore, thermal energy creates hole-electron pairs. That means, in intrinsic semiconductors, number of free electrons is equal to number of holes, In addition to current by free electrons, the current by holes ccan be explained as follows: As shown in figure above, let us consider the valence electron at L. Suppose this valence electron becomes a free electron due to thermal agitation. This will create a hole in the covalent bond at L. Since hole is a strong centre of attraction for the electrons, a valence electron at M (or from any nearby covalent-bond) may come to fill in the hole at L. ‘This wil result in the creation of hole at M. Another valence electron at NV in tum may leave its covalent bond to fill the hole at M, thus creating a hole at N. Thus, the hole having a positive charge will move from L to N ic. towards the negative terminal of supply. This will constitute hole current. Energy Band Description of Hole Current ‘The hole current can be explained easily in terms of energy ‘bands. Let us consider an electron in valence band. Due to thermal energy, this valence electron will eave the valence band and enter the conduction band as shown in the figure below. CONDUCTION BAND VALENCE BAND 10. IL 12, 1B. 15. 1, 15 This will leave a vacancy at L, thus creating a hole there. Now, the valance electron at M will come to fill the hole at L, This will result that the hole will disappear at L and appear at M. Consequently, hole will be created at N. It can bbe noted from the figure that valence electrons move along the path PML, whereas holes move along the path LMP. IMPORTANT Points Asour INTRINSIC SEMICONDUCTORS ‘When an electron is removed from a covalent bond, it leaves a vacancy behind. An electron from a neighbouring atom can move into this vacancy, leaving the neighbour with a vacancy. In this way the vacancy formed is called hole (or cotter), and can travel through the material and serve as an additional current carrier. Able is considered as a seat of positive charge, having, ‘magnitude of charge equal to that of an electron. Holes acts as virtual charge, although there is no physical ‘charge on it. Effective mass of hole is more than that of electron, Mobility of hole is less than that of electron. Free electron move in CB, while hole in VB. Tons formed are at rest or immboile. The drift velocity of electrons (y, is greater than that of holes (»). ‘A pure semiconductor is called intrinsic semiconductor. thas thermally generated current carriers. Intrinsic semiconductors have four electrons in the ‘outermost orbit of atom and atoms are held together by covalent bond. Both free electrons (n,) and holes (n,) are charge carriers in intrinsic semiconductors. 1 (in CB) =n, (in VB) For intrinsic semiconductor, the fermi energy level lies at the centre of the CB and VB. In pure semiconductor, impurity must be less than 1 in 10* parts of semiconductor, In intrinsic semiconductor, where n, = Electron density in conduction band (CB) 1n, = Hole density in valence band (VB) = Density of intrinsic carriers Because of less number of charge carriers at room temperature, intrinsic semiconductors have low conductivity. So, they have no practical use. Number of electrons reaching from valence band t0 conduction band is n= AT *¥¢ 2, where 4 is positive constant. Net charge of a pure semiconductor is zero, Physles 1.6 _ ‘A semiconductor, at room temperature, contains electrons in the conduction band and holes in the valence band, When an ‘extemal electric field is applied, the electrons move opposite to the field and the holes move in the direction of the field, thus constituting current in the same direction. The total current is the sum of the electron and hole currents. mith Hole current (i,) Electrons of covalent bond site jump from one position to another position in valence band, so hole moves opposite to the Jumping of electrons in valence band, Electron originally set fice is not involved in the process of hole motion. Motion of hole is only a convenient way of describing the actual motion of bonded electrons of valence band. Electron Current (/,) Free electrons move completely independently as conduction ‘electrons and give electron current under the effect of electric field. Direction of current i+ Electron | j flow ‘Semiconductor — Let us consider a semiconductor block of length ¢, area of cross-section A and having electron concentration n, and hole concentration n,. Across the ends of the semiconductor, an electric field £ is created by applying a battery of voltage V. This electric field is given by E Ai) sis Under the field £, the electrons and the holes both drift in opposite directions and constitute currents i, and ,,respectively, in the direction of the field. The total current flowing through the semiconductor is isiti, i,=n,edy, and ime y, where ¢is the magnitude of electron charge. i= 1,47 eA (nv, tM) or 4G netny tM) iy Let R be the resistance of the semiconductor block and p the resistivity of the block material. Then 4 a? Dividing equation (i) by equation (ii), we have 7 (Since V = iR by Ohm’s law) Ail) Substituting in it the value of /A from equation (i), we get Feay+my) ? eae on 4 ae(nceem +) ati) ‘Now mobility is defined as the drift velocity per unit field and is expressed in meter’/(volt-second). Thus, the mobilities of electron and hole are given by » = and 4, Meo FE and by = Introducing p, andy, in equation (iv), we get 1 Fm elm Het Hy 7 ) The electrical conductivity o is the reciprocal of the resistivity . Thus, the electrical conductivity of the semiconductor is given by Av) Genen= o=e(nH,+m, 1) So o=en,(u,+y,) This is the required expression. It shows that the electrical conductivity of a semiconductor depends upon the electron and hole concentrations (number densities) and their mobilities. The electron mobility is higher than the hole mobility. Temperature Dependence of Conductivity of Intrinsic Semiconductor As temperature rises, both the concentrations 7, and n, increase due to breakage of more covalent bonds. The mobilities 4, and Hy, however, slightly decrease with rise in temperature but this decrease is offset by the much greater increase in n, and n, Hence, the conductivity of an intrinsic semiconductor increases (or the resistivity decreases) with risen temperature. Thus, semiconductors hhave negative temperature coefficient of resistivity EXTRINSIC SEMICONDUCTORS ‘The conductivity of an intrinsic semiconductor depen temperature, but even at room temperature its conductivity is very low. As such, no important electronic devices can be developed using these semiconductors. Hence, thete is ‘a necessity of improving their conductivity in a controlled ‘manner. This can be done by making use of impurities. When @ small amount, say a few parts per million (ppm), of a suitable impurity is added to the pure semiconductor, the conductivity of the semiconductor is increased manifold. Such impurity added cor doped semiconductors are known as extrinsic semiconductors co impurity semiconductors. The deliberate addition of a desirable impurity is called doping and the impurity atoms are called dopants. Such semiconductors are also called doped semiconductors. The dopant has tobe such that it doesnot distort the original pure semiconductor lattice. It occupies only a very few of the original semiconductor atom sites in the crystal, A necessary condition to attain this is that the sizes of the dopant and the semiconductor atoms should be nearly the same. There are two types of dopants used in doping the tetravalent Si or Ge: (j) Pentavalent (valency 5): Arsenic (As), antimony (Sb), Phosphorous (P), etc. i) Trivalent (valency 3): Indium (In), boron (B), aluminium (Al), etc. Si or Ge belongs to the fourth group in the periodic table, therefore we choose the dopant element from nearby fifth or third group, so that the size of the dopant atom is nearly the same as that of Si or Ge. Pentavalent and trivalent dopants in Si or Ge give two entirely different types of semiconductors Lmype 2.poype n-Type Semiconductor When a pentavalent impurity atom (antimony, phosphorus or arsenic) is added to a Ge(or Si) erystal, it replaces a Ge (or Si) atom in the crystal lattice. Four of the five valence electrons of the impurity atom form covalent bonds with one each valence electron of four Ge (or Si) atoms surrounding. Is fifth valence electron is comparatively free than 4 valence electrons forming covalent bonds. At room temperature, this fifth electron gets sufficient energy and becomes free. Free 17 It should be noted that this free electron does not leave any vacancy (hole) behind it. Thus, by adding pentavalent impurity to pure Ge (or Si), the number of free electrons increases, that is, the conductivity of the crystal increases, The impure Ge (or Si) crystal is called an n-type semiconductor because it has an excess of negative charge carrier (electrons). The impurity toms are called donor atoms because they donate the conducting electrons to the crystal The fifth valence electrons of the impurity atoms occupy some discrete energy levels just below the conduction band. These are called donor levels and are only 0.01 eV below the conduction band in case of Ge, and 0.05 eV below in case of Si. Therefore, at room temperature, the fifth electrons of almost all the donot atoms are thermally excited from the donor levels into the conduction band where they move as charge carriers when an external electri field is applied Thus, at ordinary temperature, almost all the electrons in the ‘conduction band come from the donor levels only afew come from the valence band, Therefore, the main charge carriers responsible for conduction are the electrons contributed by the donors. Since the excitation from the valence band is small there are very few holes in this band, The current contribution of the holes is therefore small. Thus, in an 1-type semiconductor, the electrons are the tmajority cariers and the holes are the minority cariers. The following figure shows n-type semiconductor at T> 0K. Donor energy levels get completely empty at 300 K. (In the figure shown, its conduction band has one thermally generated electron-hole pair and 9 conduction electrons from donor atoms.) SS Comertar Oosev isi) ff Foot SS Se Se vatence bane aa Conduction band “Ep (Donor energy level] Blectronenergy IMPORTANT POINTS ABOUT A-TYPE SEMICONDUCTORS n-type semiconductors are obtained by adding a small amount of pentavalent impurity to a pure sample of semiconductor (Ge). Intrinsic + — rope op [+O — a) reirains Neutral Neutral Neutral 1. Majority charge carriers: Electrons Minority charge carriers: Hole n>rmsh>>i, Conductivity, o= ni, ¢ 1.8 4, Donor energy levels lie just below the conduction band. 5. Electrons and hole concentration: Ina doped semi- conductor, the electron concentration 1, and the hole concentration n, are not equal (as they are in an instrinsle semiconductor. It can be shown that 2 men ‘here nis the intrinsic concentration. Inann-type semiconductor, the concentration of electrons in conduction band is nearly equal to the concentration ‘of donor atoms (WV) and very lange compared to the concentration of hole in valence band, ie, n=N,>>m, ‘6. Impurity atoms, called donor atoms, are elements of V group of periodic table. 7. Net charge on n-type semiconductor is zero. ‘8 Impurity atom becomes positively charged ion. p-Type Semiconductor ‘When a trivalent impurity atom (boron, aluminium, gallium or indium) is added to a Ge (or Si) crystal, it replaces one of the Ge (or Si) atoms in the crystal lattice. Its three valence electrons form covalent bonds with one each valence electron of three Ge (or Si) atoms surrounding it. Thus, there remains an empty space, called hole. Thus, by adding trivalent impurity, number of holes increase ie., the conductivity of seminconductor increases. This impure semiconductor is called p-type because it has an excess of positive acceptor atoms because they create holes which accept electrons. ‘The impurity atoms occupy vacant discrete energy levels just above the top of the valence band. These are called acceptor levels. At room temperature, electrons are easily excited from the valence band into the acceptor levels, The corresponding holes created in the valence band are the main charge carriers in the crystal when an electric field is applied. Thus, in a p-type semiconductor, the holes are the majority carriers and the few electrons, thermally excited from the valence band into the conduction band, are minority carriers. ‘The following figure shows electron and hole concentration for p-type semiconductor at T'> 0 K. Acceptor energy levels get completely filled at 300 K. Pita rrmally generated electron-hole pair figure, one the ee ptor atoms is shown.) seven holes due to accef ———— , = Concuction band B, = 0.01 eV (Ge) 0.05 eV (Si) er area Valence band =o IMPORTANT POINTS ABOUT p-TYPE ‘SEMICONDUCTOR ‘These are obtained by adding a small amount of trivalent impurity to «pure sample of semiconductor (Ge). Neutral 1. Majority charge carriers: Holes Minority charge carriers: Electrons ny>>mgi>>i, 3. Conductivity, 6 = nyp,€ 4. pype semiconductor is also electrically neutral (not positively charged). 5. Impurity is called acceptor impurity which is element of I group of the periodic table. 6. Acceptor energy levels lie just above the valency band. 7. Electron and hole concentration: In a p-type semiconductor, the concentration of holes in valence band is nearly equal to the concentration of acceptor atoms (N,) and very large compared to the concentration of electron in conduction band, ic., n= N,>>n, 8. Net charge on p-type erystal is zero. 9. Impurity atom becomes negatively charged ion. Distinction Between Intrinsic. and Extrinsic Semiconductors EXTRINSIC SEMICONDUCTOR INSTRINSIC SEMICONDUCTOR Tt is prepared by adding a small quantity of impurity to @ pore semigonductor, such as rand _ptype semiconductors. conductor, such as pure | 1. [It is pure, natural semi- ‘Semiconductor Electronics: Materials, Devices and Simple Circuits Tn it, the two concentrations fare unequal, There is an excess of electrons in mlype semiconductors and an excess of holes in p-type semiconductors, 2, [Init the concentration of electrons and holes are ‘equal Its electrical conductivity is significantly high, Its conductivity can be controlled by adjusting the quantity of the impurity added. controlled lis conductivity also increases with temperature, but not exponentially. exponentially. with tem- perature. Distinction between n-type and p-type semiconductor [sno|_m-rvre semiconouctor | p-rvPe semiconpuctor 1. [it is am extrinsic semi [It is conductor obtained by adding pentavalent impurity to @ pure intrinsic semiconductor. ‘also an extrinsic semiconductor obtained by adding trivalent ‘impurity to @ pure intrinsic semiconductor. 2 |The impurity atoms added provide extra free electrons to the crystal lattice and are | Inte and are called acceptor called donor atoms. atoms because the created | holes accept electrons. 3. |The electrons are majority |The holes are majority carriers and the holes are | carriers and the eletrons are rinoity cartes. minority cates. | ‘The impurity atoms added create holes in the erystal ‘The hole concentration is ‘much more than the electron ‘concentration (n, >> 1). 4, |The electron concentration is much more than the hole ‘concentration (n, >> n,). Mass Action Law In semiconductors, due to thermal effect, generation of free electrons and holes takes place. ‘Apart from the process of generation, a process of recombination also occurs simultaneously, in which free electrons further recombine with holes. At equilibrium, rate of generation of charge carriers is equal to rate of recombination of charge carriers. Recombination occurs due to free electron colliding with a hole. Larger the value of n, oF n, higher is the probability of their recombination, Hence for a given semiconductor, rate of recombination « n, * n, So, rate of recombination = R (n, *,) where & = Recombination coefficient ‘The value of R remains constant fora solid, according tothe law ‘of thermodyriamics if crystalline lattice structure is same, For intrinsic semiconductor, =n, ~ So, rate of recombination = Rn? Ro,en> Rn? > ni=n,rn, (Mass action law) Under thermal equilibrium, the product of the concentration of free electrons n, and concentration of holes 1, is constant and is independent of the amount of doping by acceptor and donor impurities. ELECTRICAL CONDUCTIVITY OF EXTRINSIC SEMICONDUCTORS ‘Conducitvty of intrinsic semiconductor is given as a= (Hp, +m Hy) For n-type semiconductor, n, is negligible and n, rope = Ny ‘Similarly, for p-type semiconductor, n, * pg = OWN, Exampte 1. The energy of a photon of sodium light (= 589 nm) equals the band gap of a semiconducting material. (a) Find the minimum energy E required to create a hole-electron negligible and n, pair. (b) Find the value of E/T at a temperature of 300 K. Sou: he (@) The energy ofthe photon is E= == 1242 eV nm ‘589nm ve ‘Thus, the band gap is 2.1 eV. This is also the minimum ‘energy E required to push an electron from the valence ‘band into the conduction band. Hence, the minimum ‘energy required to create a hole-electron pair is 2.1 eV. Note: It is difficult for the thermal energy to create the hole-electron pair because of such high energy requirement ‘but a photon of light can do it easily (b) AtT=300K, KT= (8.62 x 10° eV/K) (300 K) 25.86 x 107 eV EB 21ev Thus, ir 25;86x10 Tv ~ 8! EXAMPLE 2. A p-type semiconductor has acceptor level 57 meV above the valence band, What is maximum wavelength of light required to create a hole? 10 x3x10" 57x10 x1.6x10™ 217100 A 1.40 EXAMPLE 3. A silicon specimen is made into a p-type semiconductor by doping on an average one indivnn atom per 5.x 10” silicon atoms, If the number density of atoms in the silicon specimen is $< 10 atomv/m’, find the number of acceptor atoms in silicon per eubie centimetre, ‘SOL: The doping of one indium atom in silicon semiconductor will produce one acceptor atom in p-type semiconductor: S ‘one indium atom has been dopped per 5 % 10” silicon atoms, 5x10" 5x10" 80 number density of acceptor atoms in silicon = 10" atoms/m’ = 10! atoms/cm’ EXAMPLE 4, The concentration of hole-eleetron pairs in pure silicon at 7”= 300 K is 7 10! per cubic metre. Antimony is doped into silicon in a proportion of 1 atom in 107 Si atoms. Assuming that half ofthe impurity atoms contribute electron in the conduction band, calculate the factor by which the number ‘of change carriers increases due to doping, if the number of silicon atoms per cubic metre is 5 = 10 SOL: In pure semiconductor, electron-hole pair = 7 x 10". Therefore, total charge carriers in the semiconductor, +n.= 14% 10". After doping with donor impurity, 5x10" 10” x 10" =P =25x10" > 2 $0, gg =, +7, > tyyen, 22.5108 (2 9>>n) gs Mans _ 2.5x102!—14x108 Factor = : 14x10" EXAMPLE 5. Pure Si at 300 K has equal electron (7,) and hole (n,) concentration of 1.5 * 10" mr. Doping by indium increases 1n, 03 * 10? m°, Calculate n, in the doped Si. SoL: For a doped semiconductor in thermal equilibrium, (Law of mass action) ne (15x10)? 3x10" ExamPLe 6, What will be conductance of pure silicon crystal at 300 K temperature, if electron-hole pairs per cm’ is 1.072 * 10” 2 (At this temperature, , = 1350 m*/V s and 14, = 480 cm*/V s,) Sou: o=nen,+ nen, = nell, +H) 1.14 x 10% mho/em EXAMPLE 7. A semiconductor has equal electron and hole concentration of 6 * 10¥m’. On doping with certain impurity, electron concentration increases to 9 « 10'/m’. = 75% 10m? ao Physies (i) ldentiy the new semiconductor obtained afer doping, (ii) Calculate the new hole concentration. SOL: n,= 6% LO¥im' and n, = 9% 10"? type semiconductor. (i) sna n soit af _ 36x10" (iy em 2m 9x10" = 4% 10m ExamPLe 8. Mobility of electrons in germanium of n-type is +3000 ent /volt-see and its conductivity 5 is mho/em. If effect of holes are negligible, then concentration of impurity will be (1) 8% 10" per em! 2) 9.25 * 10" per em? 3) 6 10” perem’ (4) 9% 10" per em Sot: (1) = 3900 cm?/voltsee mbho/em — 1.6x39%10 x 10!femn? EXAMPLE 9. Calculate the conductivity and the resistivity of intrinsic silicon crystal at 300 K. It is given that 1, = 1350 cmt/volt-see, u, = 480 cm?/volt-sec and the electron- hole pair concentration is 1.072 x 10" per em? at 300 K. SoL: The conductivity for intrinsic semiconductor is o=e(ny,+n,u,) Given, n, = 1.072 « 10" per em’ = 1.072 « 10" per m? 1, for intinsie semiconductor. Further, 350 em /volt-see 1350 m* /volt-see 1, = 0.048 m? / volt-see Therefore , = 1.6 x 10°" x 1.072 10" x (0.135 + 0.048) 14 x 104 mho /metre Also, 1, He =3.14 10+ siemen per metre 1 Resistivity, p 1083.14 o 185 ohm-metre EXAMPLE 10. The concentration of acceptor atoms ina p-type ‘germanium crystal is 4 * 10 per emt . Find the conductivity, of the crystal at 300 K. (1, for germanium at 300 K is 12 14, Which of the following statements is wrong? (1) Resistance of extrinsic semiconductors ean be changed as required. 2) In mype semiconductor, the number of electrons increases in valence band, (3) In paype semiconductors, the number of holes increases in valence band, (4) In pure semiconductor, fermi band is situated in between the valence band and conduction band, 15. Ina semiconducting material, the mobilities of electrons and holes are ji, and 1, respectively. Which of the following is true? Maen, Qu0 16. Ap-type semiconductor can be obtained by adding (1) arsenic to pure silicon 2) gallium to pure silicon (G) antimony to pure germanium (4) phosphorous to pure germanium 17. The valency of the impurity atom that is to be added to germanium crystal so as to make it an nype semiconductor, is aye Qs @4 3 18 The electron mobility in n-type germanium is 3900 emV-s and its conductivity is 6.24 mho/em. The impurity concentration will be nearly () 10% em? 2) 10%em* @) 10%/em? (4) 10%iem* 19, The difference in the variation of resistance with temperature in a metal and a semiconductor arises essentially due to the difference in the (1) variation of scattering mechanism with temperature (2) crystal structure (G) variation of the number of charge carriers with temperature (4) type of bond 20. A Ge specimen is doped with Al. The concentration of acceptor atoms is ~10atoms/m. Given that the intrinsic concentration of electron hole pairs is ~10°/m’, the ‘concentration of electrons in the specimen is @ 2) 10"%/m* 3) 10'/m’* (4) 10°/m? 21, In semiconductors, at room temperature, (1) the valence bands partially empty, and the conduction band is partially filled (2) the valence band is completely filled, and the conduction band is partially filled 10" /m* (2) the valence band is completely filled (4) the conduction band is completely empty 22. Ina pre silicon sample,10" atoms of phosphorus are doped per cn all he donor atoms produce carirs and 1 1200 cin? /oltse, then resistivity ofthe sample is (2) 8.2 ohm-metre (1) 5.2 ohm-metre (3) 10.4 ohm-metre (4) 2.6 ohm-metre JUNCTION DIODE (p-n JUNCTION) MA junction diode is a basic semiconductor device. It is a having acceptor impurities in one region semiconductor crystal € (p-type crystal) and donor impurities inthe other region (n-type dary between the two regions is called p.n crystal). The boun jmtion. ‘ 7 Ese 5 ’ L- Symbol Formation of p-n Junction Given diagram shows a p-n junction immediately after itis formed. p region has mobile majority holes and immobile negatively charged impurity ions. n region has mobile majority {re electrons and immobile positively charged impurity ions. Due to concentration difference, diffusion of holes starts from p tom side and diffusion of electrons starts from m to p side. jroo’ O'S" |" DOSS le’o'o'o’o’ o'e" oe" @% Oe @ loro''o'e' O'S" O'S" COX Dueto this, a layer of positive ions (in m side) and negative ions (in ‘p-side) starts to form which generates an electric field (n top side. hich opposes diffusion process. Diffusion gradually decreases ‘with the increase in strength of electric field and ultimately stops. Depletion Layer 1, The layer of immobile positive and negative ions, which has no free electrons and holes is called as depletion layer as shown in diagram. 2. Width of depletion layer is of the order of 10m, ‘As doping increases, width of depletion layer decreases. th of depletion layer 4. As temperature is increased, decreases. Semiconductor Electronics: Materials, Devices and Simple Circuits Potential Barrier at the Junction ‘Apo junction is shown in figure below. The p-type region has (positive) holes as majority change carriers, and an equal number of fixed negatively changed acceptor ions (the material ‘as a whole is thus neutral). Similarly, the n-type region has (negative) electrons as majority charge carriers, and an equal umber of fixed positively charged donor ions. Dueto separation of charges, a potential difference is developed across the depletion region called the potential barrier. Itis about (03 volt for Ge p-n junction and about 0.7 volt for silicon p-n junction It, however, depends upon the dopant concentration in the semiconductor. ‘The magnitude of the barrier electric field fora silicon junction is of the order of 81 px 10°Vm! 0 Diffusion and Drift Current Due to concentration difference, holes try to diffuse from p side ton side but due to presence of depletion layer, only those holes are able to diffuse from p to n side which have high kinetic energy. Similarly, electron of high kinetic energy also diffuse from n to p, so diffusion current flow from p to n side, 1.13 Due to thermal collision or increase in temperature, some valence electrons come in conduction band. If this occurs in depletion region, then holes move towards p side and electrons move towards 1 side, so a current is produced from n to p side. is called drift current. In steady state, both diffusion and drift current are equal and opposite. BIASING JUNCTION DIODE ‘The junction diode can be connected to an external battery in two ‘ways called forward biasing and reverse biasing of the diode. It ‘means the way of connecting emf source to p-n junction diode. It can be done in following two types: Forward Biasing ‘A junction diode is said to be forward biased when the positive terminal of the external battery is connected to the p region and the negative terminal tothe 7 region of the diode. Inthis condition, the external field, E, is directed from p towards ‘side and thus hinders the internal barrier field E.. Hence, holes in the p region and electrons in the n-region are both pushed ‘towards the junction and hence contribute to flow of current. Hole Electron (Forward Biased) The applied voltage mostly drops across the depletion region and the voltage drops across the p-side and n-side of the junction is negligible (This is because the resistance of the depletion region —aregion where there are no charges ~is very high compared to the resistance of m-side and p-side). The direction of the applied voltage (V) is opposite to the built-in potential VAs a result, the depletion layer width decreases, and the barrier height is reduced as shown in figure below. ‘The effective barrier height under forward bia is (V,~V). Ifthe applied voltage is small, the barrier potential will be reduced only slightly below the equilibrium value, and only a small number of carriers in the material—those that happen to be in the uppermost energy levels—will possess enough energy t0 1.14 ‘ross the junction, So, the current will be small, I'we increase the applied voltage significantly, the barrier height will be reduced, and more number of carriers will have the required ‘energy. Thus, the current inereases. Due to the applied voltage, electrons from n-side cross the depletion region and reach p-side (where they are minority carriers). Similarly, holes from p-side cross the junction and reach the n-side (where they are minority carriers). This process under forward bias is known as minority carrier injection, At the junetion boundary, on each side, the minority carrier concentration increases significantly compared to the locations far from the junction. Due to this concentration gradient, the injected electrons on p-side diffuse from the junction edge of p-side tothe other end of p-side. Like wise the injected holes on side diffuse from the junction edge of n-side to the other end of n-side. See figure below. pO0ed; peoee} Pp jeogot teeea} teceat t of Injected Injected electrons holes | I This motion of charged carriers on either side gives rise to ‘current. The total diode forward current is sum of hole diffusion ‘current and conventional current due to electron diffusion, ‘The magnitude of this current is usually in mA. Characteristics of forward biased p-n junction diode The circuit connections are shown in figure (a). The positive terminal of the battery is connected to the p region and the negative terminal to the n-tegion of the junction diode through a potential divider arrangement which enables to change the applied voltage. The voltage is read by a voltmeter (V) and the current by a milliammeter (mA). Starting with a low value, the forward bias voltage is increased step by.step and the corresponding forward current is noted. A graph is then plotted between voltage and current. The resulting curve OAB ‘figure (b)} is the forward characteristic ofthe diode pa @- 3 -o-r— 7 y “© BS “ S4 Ge wenn —] Hi _| B jo] lo Poreresar0s Baty yen Bes) ©) Physies In the beginning, when the applied voltage is low, the curen, through the junction diode is almost zero. Itis because ofthe potential bartier (about 0-3 V for Ge pra junction and abou, 07 V for Si junction) which opposes the applied voltage, With inctease in applied voltage, the current increases very slowly and nion-linearly until the applied voltage exceeds the potential barrie. This is represented by the portion OA of the characteristic curve. With further increase in applied voltage, the current increases very rapidly and almost linearly. Now the diode behaves as an ordinary conductor. Ths is represented by the straight line part AB of the characteristic. I this straight ine is projected back, it intersects the voltage-axis at the barrier potential voltage ImporTANT PoINTs ABOUT FORWARD BIASED DIODE 1. in forward biasing, width of depletion layer decreases, 2. Resistance offered in forward biasing, Regent = 102-252 3. Forward bias opposes the potential barrier and for > V, .2 forward current is set up across the junction. 4, Cut-in (Knee) voltage : The voltage at which the current starts to increase rapidily is known as knee voltage. For Ge itis 0.3 V and for Siit is 0.7 V. 5. Diffusion curent is greater than drift current. 6. Forward characteristic curve: 4 (aa Knee voltage x, o7 1a 2a VAvolt) 7. Forward current equation: where, 1, = Reverse saturation current V= Applied voltage T= Temperature in K K= Boltzmann's constant ‘Semiconductor Electronics: Materials, Devices and Simple Circuits Reverse Biasing ‘A junction diode is ed when the positive ‘terminal of the extemal battery is connected to the n-region and the negative terminal to the p-region of the diode as shown figure. Hole Electron tery (Reverse Biased) Inthis condition, the external field E is directed from n towards ‘p and thus aids the internal barrier field £. Hence, holes in the ‘region and electrons in the n-region are both pushed away from the junction, that is, they cannot combine at the junction, ‘Thus, there is almost no current due to flow of majority carriers, The applied voltage mostly drops across the depletion region and the direction of applied voltage is same as the direction of barrier potential. Asa result, the barrier height increases and the depletion region widens due tothe change inthe electric field. The effective barrier height under reverse bias is (V, +). See figure below. 2 1 This suppresses the flow of electrons from n to p and holes from to n. Thus, diffusion current decreases enormously compared to the diode under forward bias. The electric field direction of the junction is such that if electrons on p-side or holes on n-side in their random motion come close to the junction, they will be swept to its majority zone. This drift of carriers gives rise to current, The drift current is of the order of a few HA. Thi quite low because itis due to the motion of carriers from their minority side to their majority side across the ju The drift current is also there under forward bias, but it is negligible (WA) when compared with current due to injected carriers which is usually in mA. The diode reverse current is not very much dependent on the applied voltage. Even a small voltage is sufficient to sweep the minority carriers from one side ofthe junction to the other side of the junction. The current is not limited by the magnitude of the applied voltage but is limited due to the concentration of the minority carrier on either side of the junction. 1.15 ‘The current under reverse bias is essentially voltage independent Lup toa critical reverse bias voltage, known as breakdown voltage (V,.). When Y= V,,the reverse current of diode increases sharply. Even a slight increase in the bias voltage causes large change in the current. Ifthe reverse current is not limited, by an extemal circuit, below the rated value (specified by the manufacturer) the p-n junction will get destroyed. Once it exceeds the rated value, the diode gets destroyed due to overheating. This can hhappen even for the diode under forward bias, if the forward ccurrent exceeds the rated value. Characteristics of reverse biased p-n junction diode The circuit connections are shown in figure (a) in which the Positive terminal of the batery is connected tothe n region and the negative terminal to the p region of the junction diode. In reverse biased diode, a very small current (ofthe order of uA) flows across the junction due to the motion of the few thermally ‘generated minority carriers (electrons in p region and holes in 1 region) whose motion is aided by the applied voltage. The small reverse current remains almost constant over a sufficiently large range of reverse bias (applied voltage), increasing very litle with increasing bias. Once the breakdown voltage is, reached, the diode reverse current increases rapidly. pn “w) 1.16 Important Points ABouT REVERSE BIASED DIODE 1. Inreverse biasing, width of depletion layer increases, 2. Resistance offered in reverse biasing, Racene = 105 Reverse bias supports the potential barrier and no current flows across the juction due to the diffusion of the majority carriers. (A very small reverse current exists in the eireuit due to the drifting of minority carriers across the junction,) Breakdown voltage: Reverse voltage at which break down of semiconductor occurs is known as breakdown voltage. For Ge itis 25 V and for Si itis 35 V. Reverse characteristic curve: Vg(volt) oh The full characteristics, forward and reverse bias, are shown in figure below. The p-n junction diode, thus, is a unidirectional device. Large current (mA) flows in ‘one direction, but negligible current flows in the reverse direction. Forward resistance of an ideal p-n junction is zero. In forward bias >t y= 0 ‘on switch In other words, a forward biased p-n junction acts as short circuit. Physics ed 9, Reverse resistance of an ideal p-n junction is infinite In reverse bias thy off switch In other words, a reverse biased p-v junction acts as an ‘open cieuit. Dynamic Resistance of a Junction Diode of junction diode shows that the cument does not vary linearly with the voltage, that is, Ohm's fav is not obeyed. In such situation, a quantity known as dynamic resistance (or AC resistance) is defined. “The dynamic resistance of junction diodes defined asthe rato ofa small change in applied voltage (AV) to the corresponding small change in current (AJ), th ‘The current-voltage curve ay Rao ar Inthe forward characteristic of p-n junction diode, beyond the tuming point (knee point), however, the current varies almost linearly with voltage. In this region, R, is almost independent of V and Ohm’s law is obeyed. Breakdown of p-n Junction ‘Avalanche breakdown ‘The avalanche breakdown occurs in lightly doped junction. If the reverse bias is made very high, the minority carriers ‘acquire enough kinetic energy to break the covalent bonds near the junction, thus liberating electron-hole pairs. These charge carriers are accelerated and produce, in the same way, other electron-hole pairs. The process is cumulative and an avalanche of electron-hole pairs is produced. The reverse current then increases abruptly to a relatively large value. This is known as avalanche breakdown and may damage the junction by the excessive heat generated. The reverse bias voltage at which the reverse current increases abruptly is called the breakdown voltage. Zener breakdown Zener breakdown occurs in heavily doped junctions. In heavily doped junction, width of depletion layer is small. Even @ small reverse voltage creates large electric field. A sufficiently strong electric field manages to break the covalent bonds of the semiconductor atoms, which liberates a large number of fee minority carriers. The sudden generation of carriers rapidly increases the reverse current and this reverse bias voltage a which the reverse current increase abruptly is called the zener breakdown voltage or zener voltage. The numerical value ofthe breakdown voltage varies from tens of volts to several hundred volts depending on the number density of the impurity atoms doped into the diode. ‘Semiconductor Electronics: Materials, Devices and Simple Circuits EXAMPLE 11. A potential barrier of 0.50 V exists across a p- junction. (@) Ifthe depletion region is 5.0 « 107 m wide, what is the intensity of the electric field in this region? (b) An electron with speed 5.0 10° nvs approaches the p-n Junetion from the n-side, With what speed will it enter the psside? Sou: (@) The electri field is given as E= Vid 1.0x10° Vim Suppose the electron has a speed v, when it enters the depletion layer and v, when it comes out of it (Figure). As the potential energy increases by e * 0.50 V, from the principle of conservation of energy, 1 zm = 0504 Smt 1 oF *O.1% 10) x(5.0% 10 =1.6« 1005+ 2 rat0% og Of, 1.13 x 10=08 x 10+ (4.55 x 10) v2 => ¥,=2.7% 10' mls EXAMPLE 12. In the given figure, which of the diodes are forward biased? +10V R -10v 447 J} sve (1) 12,3 (2) 2,4,5 (3) 13,4 (4) 23,4 Sot: (2) In figures 2, 4 and 5, p-side reside EXAMPLE 13. What is the current in the circuit shown below? -4V Pn 3000 4y 9 more positive as compared to (0a 2) 107A Q)1A (4) 0.104 Sou: (1) The potential of p-side is more negative than that of n-side Hence, diode is in reverse biasing. In reverse biasing, diode acts as open circuit, hence no current flows. EXAMPLE 14, The I-V characteristic of a p-n junction diode is shown in figure. Find the approximate dynamic resistance of the pm junction when (a) a forward bias of 1 volt is applied, (b) a forward bias of 2 volts is applied, (ma) Tz za Pwo) Sot: . (2) The current at | volt is 10 mA and at 1.2 volt itis 15 ma. ‘The dynamic resistance in tis region is Ret 0.2volt ai SmA (©) The current at 2 volt is 400 mA and at 2.1 volt it is 800 mA. The dynamic resistance in the region is Av 20 volt © Ai 800-400 400mA 15-10 =0.252 EXAMPLE 15. Figure shows a diode connected to an external resistance and an emf, Assuming that the potential drop across the diode is 0.5 V, obtain the value of current in the circuit in milliampere. Sou: = 4.5 V,R= 1002 Voltage drop across p-n junction = 0.5 V Effective voltage across resistor, Y= 4.5 ~ 0- 100. Current in the cireuit, = = 0,04 « 1000 mA = 40 mA EXAMPLE 16. A.2 V battery may be connected across the points 4 and B as shown in figure. Assume that the resistance of ‘each diode is zero in forward bias and infinity in reverse bias, find the current supplied by the battery if the positive terminal ‘of the battery is connected to (a) the point 4 (b) the point B. D, loa Sot: (a) When the positive terminal of the battery is connected to the point A, the diode D, is forward-biased and D, is reverse-biased. The resistance ofthe diode D, is zero, and it can be replaced by a resistanceless wie. Similarly, the resistance of the diode D, is infinity, and it can be replaced by a broken wire. The equivalent circuit is shown in figure ‘The current supplied by the battery is 2 V/10 = 0.2.A. woo 20a he i Ly + @) (o) (b) When the positive terminal of the battery is connected to the point B, the diode D, is forward-biased and D, is reverse-biased. The equivalent circuit is shown in figure (b), The current through the battery is 2 V/202=0.1 A. ExampLe 17. What are the readings of the ammeters , and 4, shown in figure? Neglect the resistance ofthe meters. 2 (Ay |_|} —_— 2v loa L_@—iti i es series with ammeter A, 18 reverse ove Therefor, "tis zero, Reading of A, is zero and nse reading of A, is 0-2 8, Calculate th start eonductin i. 1fthe Ge ues of Y, and? ant through Ge diode. Drop across Ge value of V,and/ ifthe Si diode ang at 0.7 ¥ and 03 V, respectively, sfiode connection is reversed, what ExampLe 1 the Ge diode inthe given circuit will be the new val Sou: Conduction will st diode is 0.3 V. Hence, y= 12- inthe oad is 03=1L7V ‘The current i nve Se MT 9 34mA R, 5K2 ‘On reversing the connections ‘biased and conducts no ‘current. The effective forward voltage across Ris ‘This will appear as output, that is, V=u3V “The current in the load is of Ge diode, it will be reverse- Only Si diode will conduct. 12V-0.7V=113¥. p-n JUNCTION lene Ce Ueda aus "An electronic device which converts alternating current/voltage {nto direct current/voltage is called rectifier. ‘A pon junction diode offers a low resistance for the current to flow when forward-biased, but a very high resistance when reverse-biased. It, thus, passes current only in one direction and acts as a rectifier. The junction diode can be used either as an half-wave rectifier when it allows current only during the positive half-cycles of the input AC supply, or as a full-wave rectifier when it allows current in the same direction for both half cycles of the input alternating current. Half-wave Rectifier The half-wave rectifier circuit is shown in figure (a) and the input and output waveforms in figure (b). The alternating input voltage is applied across the primary P,P, of a transformer. 5\S; isthe scone ofthe sane transformer 5, is connected eo crystal of the junction diode and S, is connected (0 type crystal through a load resistance ‘Semiconductor Electronics: Materials, Devices and Simple Circuits 1.19 ‘Transformer R i ———— i g i Output Volta, "Fluctuating DC 7 © During the first half-cycle of the AC input, when the terminal ‘5, of the secondary is positive and S, is negative, the junction diode is forward biased. Hence, it conducts and current flows through the load 2, The current produces an output voltage of the same shape as the half-cycle of the input voltage across the load. During the second half-cycle of the AC input, the terminal S, isnegative and S, is positive. The diode is now reverse biased. Hence, there is almost zero current and zero output voltage across R,. The process is repeated. Thus, the output current is unidirectional, but pulsating, as shown in figure (b). Since the output current corresponds to one half of the input voltage wave, the other half being missing, the process is called half-wave rectification. iuportant Points About HALF-Wave RECTIFIER 1. During positive half cycle, diode is forward biased and hence output signal is obtained. 2. During negative half cycle, diode is reverse biased and hhence output signal is not obtained. 3. Output voltage is obtained across the load resistance R,. Itis not constant but pulsating (mixture of AC and DC) in nature. 4. Average output in one cyle, he rt, (r,= Forward biased resistance) 5. RMS output, 6. The ripple frequency («for half wave rectifier is same as that of AC. Center Tap Full-wave Rectifier Ina full-wave rectifier, a unidirectional, pulsating output current -d for both halves of the alternating input voltage. ly, it requires two junction diodes so connected that ‘one diode rectifies one half and the second diode rectifies the second half of the input. o ‘The circuit for a ful input and output waveforms in figure (b). The alternating input voltage is applied across the primary P,P, of a transformer. The terminals S, and S, of the secondary are connected tothe p-type crystals ofthe junction diodes D, and D, whose n-type crystals are connected to each other. A load resistance R, is connected across the n-type crystals and the central-tap T of the secondary fave rectifier is shown in figure (a) and the a AC Signal Output signal Fluctuating DC ® During the first half-cycle of the AC input voltage, the terminal S, is suppose positive relative to T and S, is negative. In this situation, the junction diode D, is forwatd biased and D, is reverse biased. Therefore, D, conducts while conventional current flows through diode D,, load R, and the upper half ofthe secondary winding, as shown by solid arrows. During the second half-cycle of the input voltage, S, is negative relative to 7 and S, is positive. Now, D, is reverse-biased and does not conduct while D, is forward-biased and conducts. The ‘current now flows through D,, load R, and the lower half of the secondary, as shown by dotted arrows. It may be seen thatthe current in the load R, flows in the same direetion for both half- cycles of the alternating input voltage. Thus, the output current is a continuous series of unidirectional pulses. However, it ean bbe made fairly steady by means of smoothing filters. does not. The IMPORTANT PoINTs AgouT FULL-WAVE RECTIFIER 1, During positive half eycle, diode D, is Forward biased and D, is reverse biased. Output signal is obtained due to D, only. 2. During negative half cycle, diode D, is reverse biased and Dy i forward biased, Output signal is obtained due to D, only. 3. Output voltage is obtained across the load resistance f. tis not constant but pulsating in nature 4. Average output 5. RMS output :V,.,= 6. Ripple frequency: The ripple frequency of full wave rectifier = 2 * (Frequency of input AC) Full-wave Bridge Rectifier Four diodes D,, D,, D, and D, are used in the circuit. Output is taken across load resistor R,. During positive half cycle D, and D, are forward biased and D, and D, are reverse biased. During negative half cycle D, and D, are forward biased and D, and D, are reverse biased. Alternating | supply Input altemating signal “The rectified voltage is in the form of pulses ofthe shape of half sinusoids. Though it is unidirectional, it does not have a steady value. To get steady DC output from the pulsating voltage normally a capacitor is connected across the output terminals (paralel tothe load R,). One can also use an inductor in series with R, for the same purpose. Since these additional cr filter out the AC ripple and give a pure DC voltage, so they are called filters. 1. Capacitor Filter acepe foun ofrecer 2.L-C Filter Ace [Owpuco eciier 3. Filter (Best Filter) DC Jw ac inusoidal voltage of ‘ampli is half wave rectifier. No fit frequency 50 Hz is applied to @ Ee reed the Yad resistor i 1000 2. The forward resistance, ideal diode is 10 2. Calculate () peak, average and rms values of load current i) DC power output Sou: Gi) P= Lo x R, = (7.88 * 10°F * 10? = 62 mW ExampLe 20. The half wave rectifier supplies power to a 1 KQ load. The input supply voltage is 220 V. Neglecting forward resistance of the diode, calculate ON, Gi) 4, Sou: Vy _ N2V gg _V2_x 220 fi) Vi_ = te = eles = 2 OO ng OMe ae 314 a ¥, @ le Bee mA. EXAMPLE 21. A full wave rectifier supplies a load of 1 k2. ‘The AC voltage applied to the diodes is 220 volt rms. If diode resistance is neglected, calculate () average DC voltage i) average DC current Sou: (i) Average DC voltage, ¥, = = 0.636 ¥, z where, V, = Maximum voltage across each half of the secondary winding. If V be the rms voltage across each half of the secondary winding, then 7 0.636 « V: 9 x 220 = 198 volt Semiconductor Electronics: Materials, Devices and Simple Circuits (i) For full wave rectifier, Vag _ 198 FE = Joop 98 mA R, 1000 EXAMPLE 22. A full wave p-n diode rectifier use load resistor of 1500 No filter is used. Assume each diode to have idealized characteristic with R= 10.Q and R= o. The voltage applied to each diode has amplitude of 30 volts and frequency 50 Hz. Calculate (@) peak, DC and rms load current Gil) DC power input (ii) AC power input Sou: () Peak current, 7, 30 10 +1500 DC load current, /, = 0.636 1, 14.= 0.636 * 19.9 mA = 12.66 mA 1, =14x10° A la IE (ii) DC power output, Py=lexR, = (12.66 x 10° « 1500= 240.41 mW (iii) AC power input, PHI, (R+R) = (14 « 10°) (10 + 1500) = 295.96 mw SOME SPECIAL DIODES Zener Diode Zener diode is a reverse biased heavily doped semiconductor (silicon or germanium) p-n junction diode, which is operated exclusively in the breakdown region. The symbol of a zener diode is shown in figure below. — For normal operation of a zener diode, in breakdown region, the current through the diode should be limited by an external ireuit. Hence, the power dissipated across the junction is within its power-handling capacity. Unless this precaution is observed, 4 large current will destroy the diode. The ¥-1 characteristic curve for the zener diode is shown in figure below. 421 Forward characterisic Reverse characteristic curve It-can be seen from the figure that as the reverse voltage applied to the p-n junction is increased, at a particular voltage, the ‘current increases enormously from its normal cut off value. This voltage is called zener voltage or breakdown voltage (V,). Zener diode as Voltage Regulator In reverse breakdown region of the reverse characteristic curve, the voltage across the diode remains almost constant fora large range of currents. Hence, the diode may be used to stabilize voltage at a pre-determined value. It can be designed by properly controlled doping of the diode, to stabilize voltage at any desired value between 4-100 volt. Figure below shows a simple circuit for stabilizing voltage across a load 2, Rs + Fluctuating input ‘The circuit consists of a series voltage dropping resistance R, and a zener diode in parallel with the load R,. The zener diode is selected with a zener voltage V, equal to the voltage desired across the load. The fluctuating DC input voltage may be the DC ‘output of a rectifier. Whenever the input voltage increases, the excess voltage is dropped across the resistance R,. This causes an increase in the input current. This increase is conducted by the zener diode, while the current through the load and hence the voltage across it remains constant at V Likewise, a decrease in the input voltage causes a decrease in the input current. The current through the diode decreases correspondingly, again ‘maintaining the current through the load constant. Since the resistance R, absorbs the input voltage fluctuations to give a constant output voltage V, the cireut cannot work if the input voltage falls below V, Photodiode A photodiode is a reverse-biased pn junction made from a photosensitive semiconductor. ‘The junction is embedded in clear plastic. The upper surface across the junction is open to 1.22 light, while the remaining sides of the plastic are painted black ‘orenclosed in a metallic case. The entire unit is extremely small, of the order of a 0-1 inch in size Light NS R A P ‘When no light is falling on the junction and the reverse bias is of the order of a few tenths of a volt, an almost constant ‘small current (mA) is obtained. This dark current is the reverse saturation current due to the thermally-generated minority- carriers (electrons in p-region and holes in n-region). When light of appropriate frequency is incident on the junction, additional electron-hole pairs are created near the junction (due to breaking of covalent bonds). These light generated minority- carriers cross the (reverse biased) junction and contribute to the (reverse) current due to thermally-generated carriers. Therefore, the current in the circuit increases (a fraction of a mA). This, so- called photoconductive current varies almost linearly with the incident light flux. ‘The p-n photodiodes can operate at frequencies ofthe order of 1 MHz. Hence, they are used in high-speed reading of computer punched cards, light-detection systems, light-operated switches, electronic counters, etc. Photodiode is always reverse biased due to following reasons: 1. In a forward bias p-n junction, the width of depletion region is less and keeps on decreasing as we increase voltage. So, there isa small area where photons will break the bonds and less current is generated, Whereas in reverse bias p-n junction, the width of depletion region is more and keeps on increasing as we increase voltage. So, the area for photons to work on is more and the large current can be generated. 2. If the diode is forward biased, it will conduct a lot of current and you can't detect the small amount of excess ‘current produced by the photoelectric effect. When the diode is reversed biased, very small current flows and the detection of the photocurrent is much easier. ‘One may understand this in following manner. When light is incident on p-n junction additional current cariers (lectron-hole pairs) are generated. Forward bias p-n junction has already lot of current carriers. Therefore, fractional increase in current carriers in itis negligible. On the other hand, reverse bias p-n junction has very small number of cusrent carriers, therefore fractional increase in ccurrent carriers is easily detected. ht-Emitting Diode (LED) ‘When a p-n junction diode is forward-biased, both the electrons ‘and the holes move towards the junction, As they cross the Physics junction, the electrons fall into the holes (ie. recombine) Hence, chergy is released atthe junction (because the electrons {all from a higher to a lower energy level). Incase of Ge and Sj diodes, the energy released is infrared radiation. If, however, the diede is made of gallium arsenide or indium phosphide, the nergy released is visible ight. Te diode is then alld afghe emitting diode (LED). Light LEDs have replaced incandescent lamps in many applications tease of their low input power, long life and fast onoff ‘switching. ‘They are extensivel calculators, etc. ly used in fancy electronic devices like Solar Cell 1, Solar cell or Photovoltaic cell is a device which converts light energy into electrical energy directly. A Solar cell is a speci ‘pon junction diode fabricated with ‘transparent window to allow light to fall on the diode. ‘When the p-n junction is illuminated with light (photons) with energy (iv) greater than the energy gap (E,) of the semiconductor, then electron-hole pairs are generated due to the absorption of photons. The junction area is kept ‘much larger for solar radiation to be incident because we are interested in more power. 2. A simple p-n junction solar cell is shown in figure below. ‘A p-Si wafer of about 300 ym is taken over which a thin layer (-0.3 um) of n-Si is grown on one-side by diffusion process. The other side of p-Si is coated with a metal (back contact). On the top of n-Si layer, metal finger electrode (or metallic grid) is deposited. This acts as a front contact. ‘The metallic grid occupies only a very small fraction ofthe cell area (<15%) so that light can be incident on the cell from the top. Semiconductor Electronics: Materials, Devices and Simple Circults 1.23 3. The diode is fabricated such thatthe generation of electron- hole pairs takes place in or near the depletion region of the diode. Due to electric field of the junction, electrons and holes are separated before they recombine. The direction of the electric field is such that electrons reach n-side and holes reach p-side. Electrons are collected on n-side and holes are collected on p-side giving rise to an emf, The electrons reaching the n-side are collected by the front contact and holes reaching p-side are collected by the back contact. Thus, p-side becomes positive and n-side becomes negative giving tise to photovoltaic emf. 4. When an extemal load is connected, as shown in the figure below, a photocurrent / flows through the load. The magnitude of the photocurrent depends on the intensity of incident light (photocurrent is proportional to incident light intensity), AY Deter 5. Atypical -V characteristics curve of a solar cell is shown in the figure below. 1 ~ V characteristics of solar cell is drawn in the fourth quadrant of the coordinate axes. his is because a solar cell does not draw current but supplies the same to the load, 6. Semiconductors with band gap close to 1.5 eV are ideal materials for solar cell fabrication. Solar cells are made with semiconductors like Si (E, = 1.1 eV), GaAs(E, = 1.43 eV), CéTe (E, = 1.45 eV), CulnSe2 (E, = 1.04eV), ete. The important criteria for the selection of a material for solar cell fabrication are (i) band gap (~1.0 to 1.8 eV), (i) high ‘optical absorption (~104 em), ii electrical conductivity, (iv) availability of the raw material, and (v) cost, Not that sunlight is not always required for a solar cell, Any light ‘with photon energies greater than the bandgap will do. 7. Solar cells are used to power electronic devices in satellites and space vehicles and as power supply to some calculators. Varlable Capacitor (Varactor) It can be used as a capacitor. Here depletion layer acts as, dielectric material and remaining p and n parts act as metallic Its symbol is i Itis interesting form of LED in which special construction helps to produce stimulated radiation as in laser. EXAMPLE 23. A zener diode of voltage V7 (= 6 Volt) is used to ‘maintain a constant voltage across a load resistance R, (=1000 ) by using a series resistance R, (=100 Q). If the emf. or source is E (= 9 V), calculate the value of current through series resistance, zener diode and load resistance. What is the power ‘being dissipated in zener diode? Sou: Here, E=9 V; V,=6 V; R, = 1000 Q and R,=100 2, Potential drop across series resistor decrease one is, V=E-V,=9-6=3V Current through series resistance R, is v3 === = 003A Rs 100 ‘Current through load resistance R, is Current through zener diode is 1,= 1-1, = 0.03 - 0.006 = 0.024. Power dissipated in zener diode is P,=V,1,= 6 * 0.024 = 0.144 watt EXAMPLE 24, A zener diode is specified having a breakdown voltage of 9.1 V with a maximum power dissipation of 364 mW. ‘What is the maximum current that the diode can handle? SOL: Maximum current that the given diode can handle is 364x107 Aie, 40 mA, OL 1.26 20, The potential barrier in the depletion layer is due to (1) ions 2) holes @) electrons (4) forbidden band 21, Ina forward biased p-n junction, the potential barier (1) decreases: (3) remains constant (2) increases (4) becomes zero 22, Inmaype semiconductors, majority charge carriers are (1) holes (2) protons (3) neutrons (4) electrons 23, In the given circuit, resistance of resistor is 1 kQ and threshold (knee) voltage for diode is 0.7 V. vs Vu Correct variation of V,, with respect to V, is Vo a 07 2) o7 @) (4) 07 Ruedas “Transistor was invented by William Bradford Shockley, John Bardeen and Walter Houser Brattain ‘Transistor is a three terminal device which transfers a signal Tet cstance circuit to high resistance circuit. Its formed ven ath ‘of one type of extrinsic semiconductor (p or n By a awhed between two thick layers of other two same type of extrinsic semiconductor. Each transistor has three terminals: (i Emitter (ii) Base (ii) Colleetor Emitter It emits the majority and medium in size. carriers towards base. It is highly doped sh is sandwiched by emitter Itis the middle part of transistor whic! ‘ (E) and collector (C). Itis lightly doped and very thin in size. Collector Itis that part of transistor which collects the majority carriers which are emitted by emitter. It has large size and is moderately doped. “There are two semiconductor junctions in transistor: (j) The junction between emitter and base is known as emitter. base junction (,)- (i) The junction between base and collecter is known as base- collector junction (Jy). Transistors Types © mp-n transistor If a thin layer of p-type semiconductor is sandwiched between two thick layers of n-type semiconductor, then ‘the transistor formed is known as n-p-n transistor. pee * E ct ' EG LS © pon-p transistor Ifa thin layer of n-type of semiconductor is sandwiched between two thick layer of p-type semiconductor, then the transistor formed is known as p-n-p transistor. ORD LS a pp Semiconductor Electronics: Materials, Devices and Simple Circults Working of n-p-n transistor ‘The emitter base junction is forward bias and collector base junction is reversed biased of n-p-n transistor in circuit (A) and symbolic representation is shown in figure (B), 4127 Emitter-base Collector-base “junction, eduction n p yn = & E = Tt Ic | it [Bese =| |} q ) ma? F mA ten Oven 1 aft = yf =e ae eH (A) ke Neck B Ts} ae a f Hf (8) When emitter base junction is forward biased, electrons (majority carriers) in emitter are repelled toward base. The barrier of emitter base junction is reduced and the electrons enter the base. About 5% of these electrons recombine with hole in base region resulting in small current (,. The remaining electrons (= 95%) enter the collector region because they are attracted towards the positive terminal of battery resulting in collecter current (.). For each electron entering the positive terminal of the battery connected with collector base junction, an electron from negative terminal of the battery connected with emitter base junction leaves it. ‘The emitter current (/,) is more than the collector (,). The base current is the difference between J, and J; and Proportional to the number of electron hole recombination in the base. Working of p-n-p Transistor ‘When emitter-base junction is forward biased, holes (majority carriers) in the emitter are repelled towards the base and diffuse through the emitter base junction. The barrier potential of ¢mitter-base junction decreases and holes enter the n-region, i.e, base. Asmall number of holes (~ 5%) combine with electron of base-region resulting small current (/,). The remaining holes (= 95%) enter into the collector region because they are attracted towards negative terminal of the battery connected with collector-base junction. These holes constitute the collector current (/,). Asone hole reaches the collector, itis neutralized by the electron coming from the battery. As soon as one electron and a hole is neutralized in collector, a covalent bond is broken in emitter region. The electron hole pair is produced. The released electron enter the positive terminal of battery and hole moves towards the collector. So, I, +I, Note: n-p-n transistor is preferred over p-n-p transistor as electrons have more mobilty than that of holes. IMPORTANT POINTS REGARDING TRANSISTOR 1, The collector region is made physically larger than the ‘emitter because collector has to dissipate much greater power. 2, Transistor mostly works in active region in electronic devices. In this region, it works as an amplifier. 3. Transistor, ie, it isa short form of two words “Transfer resistor”. Signal is introduced at low resistance circuit and output is taken from high resistance circuit. 4. Base is lightly doped. Otherwise, most of the charge cartiers from the emitter would have recombined in base region and did not reach the collector. 5. Transistor is a current operated device, ie, the action of transistor is controlled by the motion of charge carriers, ive, current. 1.28 RISO SSN Ae) CLs Led ‘A transistor can be connected in a cireuit in the following three different configurations: (a) Common-base configuration (b) Common-emitter configuration (©) Common-collector configuration aspen ourPuT (@) COMMON-BASE nsp-n c E ourPuT B input 1 =H (6) COMMON-EMITTER (@) COMMON-COLLECTOR CHARACTERISTICS OF BE Meig Common Emitter Characteristics Circuit diagram Taput Output Common emitter mp-n transistor Physics Input characteristics obtained by plotting the bas characteristics are rbtaine . 1 ct ta a os taflector-emitter voltage (Hee) 3 In aN °/s (WA) e/e Y 4 0 Vg (Volts) in yp, With increase in 7 with inorease in Yop ay, 2 Ty cen Ms delayed as most ofthe carers are pul by collector. 3. These character biased junction diode. resemble with those of a forward Output characteristics 1. The output characteritis are obtained by plotting collector secon versus collector-emitter voltage (V,) at constant value of base current (J,) T. 43 eR (mA) Active region [= 40 mA ‘Saturation re Cut off region 30 40 30 19 20 60 V,, (Volts) 2. I; increases with increase of Vg, upto 1 vott and beyond 1 volt it becomes almost constant. 3, The value of F, upto which J inereases is called the ke? voltage. 4, Above knee voltage, J. is almost constant. 5, The region for V., < 1 volt is called saturation region both emitter and collector are forward biased. 6. In the region J, £0, both emitter and collector are reve" biased and it is called the cut-ff region. 7. The central region, where the curves are uniformly space and sloped, is called the active region. In this e8% the emitter is forward biased and the collector is F'°™* biased. Semiconductor Electronics: Materials, Devices and Simple Circuits 129 Figure below shows typical output voltage (¥,) - input voltage ewrrersunenion | “COLLECTOR | TRANSISTOR BIAS suncrion Bias | operation | (V) characteristic, called the transfer characteristic of the base ‘Forward Reverse ~ biased transistor. It has three well-defined regions as follows: Forward Forward Saturation = 0h=0 (et Reverse R o see Sof Cutof Active Transfer characteristic Uf ee eee 1vis a graph showing the variation of collector current J with base current J, at constant collector-emitter voltage Vip. AS shown in fgute the transfer characteristic of transigor is almost a straight line, T-(mA) 10 0 100 200-7, (uA) ‘Transfer characteristic of CE n-p-n transistor. THREE STATES OF A TRANSISTOR To understand the operation of a transistor asa switch, we frst study the three states or conditions in which a transistor can work. Figure shows the circuit diagram of a base-biased n-p-n ‘transistor in CE configuration, Here R, isa resistor in the input circuit and R. in the output circuit. Applying Ksichhoff's rule to the input and output circuits separately, we get Vag y+ Vag and Va" Tet Veg Veg Veo ‘The voltage V,, can be regarded as the DC input voltage V, and Veq4s the DC output voltage V, So we can write Vm ERy* Vag and y= Veo 1R 0) Tange fc Saturation Tegion Cut off region Refer figure above, When J, increases from zero to a low value (less than 0.6 V in case of a Si transistor), the forward bias ofthe emitter-base junction is insufficient to start a forward current. That is, ,=0 and hence /_=0. The transistor is said to be in the cut off region. From equation (1), the output voltage V,= Vx. Active region When Y, increases slightly above 0.6 V, a current /, flows in the output circuit and the transistor is said to be in the active state, From equation (1), as the term JR, increases, the output voltage V, decreases. Now as V; increases, /, increases almost linearly and so V, decreases linearly tll its value becomes less than 1.0 V. Saturation region When J; is high ic. the emitter base junction is heavily forward biased, a large collector current /, flows which produces such A large potential drop across load resistance R. thatthe emiter- collector junction also gets forward biased. The output voltage ¥, decreases to almost zero. The transistor is said to be in the saturation state because it cannot pass any more collector current J. Obviously, the characteristics from cut off state to active state and from active stage to saturation state are not sharply defined because these regions of the transfer characteristic are non- linear. TRANSISTOR AS A SWITCH Digital devices like computers perform millions of switching ‘operations every day. Transistors can be used as such swift switches in computer citcuits ‘Transistors have many advantages over other electrically ‘operated switches such as relays 1. Transistors are small, cheap and reliable. ‘They have no moving parts. ‘They have long life in well-designed circuits. ‘They can switch on and off millions of times a second. 1.30 Switching Action of a Transistor A transistor ean be used asa switch if it is operated in its eut of and saturation states only. A switch circuit is designed in such ‘a manner that the transistor does not remain in the active state, As Tong as the input voltage is low and unable to forward-bias the transistor, the output voltage V, (at V,.) is high. I Vis high enough to drive the transistor into saturation, then Vis low, nearly zero. When the transistor is not conducting, it is said to be switched off and when itis driven into saturation, itis said to be switched on, So if we define low (0) and high (1) stages as below and above certain voltage levels corresponding to cut ‘off and saturation of the transistor, then a low input switches the transistor off and a high input switches it on. Alternatively, we can say that a low input to the transistor gives a high output and high input gives a low output eeNesakom cise An amplifier is a circuit (consisting of at least one transistor) which is used for increasing the voltage, current or power of alternating form. Amplifying means to increase the size or to magnify an input signal. The output signal of an amplifier is an enlarged version of the input signal. Power supply V, St | Amplifier Ho 1 input L JF Ouput voltage * * voltage Figure illustrates the general concept of an amplifier. An amplifier is provided with (i) two input terminals forthe signal to be amplified, (ii) two output terminals for connecting the load and (iii) a means of supplying power to the amplifier. One input terminal and one output terminal (shown as earthed) are common. AC Voltage Gain (A,) ‘The usefulness of an amplifier is expressed in terms of the gain of the amplifier. The AC voltage gain of an amplifier is defined ‘as the ratio of the change in the output voltage (AV,) to the corresponding change in the input voltage (AV). Thus, Physics It may be noted that only the AC and not the DC components of the input and the output voltages are used to calculate the voltage gain BLO L er uranium (COMMON EMITTER) Figure shows a common-emitter amplifier circuit using an pan transistor. The emitter is made common to the input and the output circuits “The input (base-emitter) circuit is forward-biased by a low. voltage battery Vyy 80 thatthe resistance of the input circuit is lector-emitter) circuit is reverse-biased by small. The output (coll means of a high voltage battery V.. $0 that the resistance of the output circuit is high. R, is a load resistance connected in the collector-emitter output circuit. The weak input AC signal is applied acros the base-emitter circuit and the amplified output signal is obtained across the collector-emitter circuit Leti,j,and ibe the emitter-current, base-current and collector- current, respecitively, when no AC voltage signal is applied to the input curcuit. The arrows represent the direction of the hole current, that is conventional current which is opposite to the direction of electron current. By Kirchhoff’ fist law, we have Due to the collector current i. (which is only slightly smaller that i,), the voltage drop across R, is i.R,. Therefore, the collector -to -emitter voltage (potential between collector and emitter) Vig, would be given by Veg Veo ic @ When the input AC voltage signal is applied across the base emitter circuit, it changes the base-emitter voltage and hence the emitter-current i, which in turn changes the collector current i Consequently, the collector-to-emitter voltage Vi, varies in accordance with equation (2). This variation in V/,, when the input signal is applied , appears as an amplidied output. Working of Amplifier ‘The amplifying action of a transistor can be explained 2 follows ‘Semiconductor Electronics: Materials, Devices and Simple Circuits 1, When ac signal is not applied, the base current is available in small quantity in microamperes, which is represented by OP and the corresponding collector current in milli- amperes is represented by PO. 2. When ac signal is applied, the potential difference between the base and emitter changes continuously. Base current (1,) increases from OP to OA, then decreases from OA to OP, then decreases from OP to OB, and once again increases from OB to OP. 3. This variation in base current is reflected in the collector current as shown in figure above. The collector current (7) increases from PQ to AA,, falls from A4, to BB, and again increases from BB, to PQ. Thus, a variation in the base current in micro-amperes produces a corresponding variation in the collector current in milli-amperes. 4, Here a weak input signal (J, in mA) in base emitter circuit results in amplified output signal (J. in mA) in collector emitter circuit. Hence, transistor in this case acts as an amplifier. Phase Relationship between Output and Input of Common Emitter Transistor During positive half cycle of the input, forward-bias of base- ‘emitter junction increases. This increases the emitter current (,) and hence the collector current (/,). In consequence, the voltage drop across the load resistance R, increases. From equation Q, it follows that V, decreases below the normal value. So, Positive cycle of input results in negative cycle of output. During negative half cycle of the input sinusoidal signal, forward-bias of base-emitter junction decreases. This decreases the emitter current and hence the collector current. In consequence, the voltage drop across the load resistance R, decreases. From equation (2), it follows that V. increases above the normal value. So, negative cycle of input results in positive cycle of ‘output. Hence in a common emitter amplifier, output is out of phase by 180° than input. 1.31 Current Gains in a Transistor Usually two types of current gains are defined fora transistor: () Common base current amplification factor or AC current gain a: It is defined as the ratio of the small change in the collector current to the small change in the emitter current when the collector base voltage is kept constant. Thus, where c,, = DC current gain of common base amplifier. Common emitter current amplification factor or AC current gain f: It is defined as the ratio of the change in collector current to the small change in base current at constant collector-emitter voltage (V,.) when the transistor is in the active state. di) also known as small signal current gain and its value is very large. The direct ratio of J. and J, gives the DC current gain (B,) of the transistor. Hence, (iii) AC voltage gain: Suppose, on applying an AC input voltage signal, the input base-current changes by A/, and correspondingly the output collector-current changes by Al. If R,, and R,,,be the resistance of the input and the output circuits, respectively , then. AMX Rag Alyx R, Aly Ry Now, AJ, /Al, is the AC current gain (AC) and R,,,/ Ry is the resistance gain, +4, B,, » Resistance gain AC power gain : It is defined asthe ratio of the change in the output power to the change in the input power. Since, Power = Current x Voltage +. AC power gain = AC current gain AC voltage gain =B,*4y =B.x (B, * Resistance gain} =P, x Resistance gain (v) Transconductanee: It is defined as the ratio of the small change in the collector current to the small change in the emitter-base voltage. It is denoted by g,, Thus, (iv) ee ol) = = fy = BR (From 1) ‘The transconductance is also called transfer conductance ‘and has the same units of ‘conductance (siemen or mho). The transconductance depends on the geometry, doping levels and biasing of the transistor. Relation Between a and B For both mp-n and p--p transistors, we have Iga hyt le For small changes, we can write AI,= Al, + Ale Dividing both sides by A/., and Beira As the value of [, is about 15% of J, of 1 i 95-99% of Ip «is about 0.95 to 0.99 and B is about 20 to 100. The CE configuration is frequently used as it gives high current gxin as well as voltage gain. ExampLe 26. The value of B for a transistor for which = 0.95, will be aig Qa @) 19 (4) 0.19 Sot: (1) 0.95 @ 7 9 BT 71-095 EXAMPLE 27. In a transistor, the value of fis 50. Calculate the value of 0. Sou: p=2 950-8 9 50-Sda=a I-a ExampLe 28. Calcula 1,720 wAand P= 100 Sot: /,~Bl,= 10" I mon emitter amplifier, current gain ‘mA, calculate the collector when transistor is EXAMPLE 29. For @ co! is 50, If the emitter current is 6.6 vs base current Also calualte current gai, working as common ‘base amplifier. Ai) Using equation (i), we eet 6.6 = 50 f, + 1p= 5M = |, s =0.129mA 66 =50x 5° =6.47mA 1.250% Hence, 63 1+B SI ‘Transistor with | 15 is connected to common- it will be the maximum collector current = 098 And, EXAMPLE 30. ‘base configuration. Wha for an emitter current of 5 mA? Sot: B=75,1,=5mA 5 _75 EXAMPLE 31. In n-p-n transistor circuit, the collector current ig 10 mA. 1f 95% of the electrons emitted reaches the collector, ‘what is the base current? SoL: [,= 95% I,= 0.95 I, 100 095 95 x 10 mA= 10.53 mA Now, Ip =Jo+ ly 10,53 10= 0.53 mA >i EXAMPLE 32. A p-n-p transistor is used in common-emitter mode in an amplifier circuit. A change of 40 mA in the base current brings a change of 2 mA in collector current and 0.04 v in base-emitter voltage. Find the (input resistance (R,) ‘Semiconductor Electronics: Materials, Devices and Simple Circuits (ji) the base current amplification factor (8) }) voltage gain of the amplifier, if load of 6 KGL is used 109A AV, = 0.04 volt, R, = 6 K2=6 x 10° (@ Input resistance, = 10 2=1kQ (ii) Current amplification factor, B= Ale _ 2x 10" - AI, 40x10 (iii) Voltage gain in common emitter configuration, R 6x10° Re Teor = 300 ExamPLe 33. A transistor is used in common-emitter mode in an amplifier cireuit. When a signal of 20 mV is adéed to the base-emitter voltage, the base current changes by 20 WA and the collector current changes by 2 mA. The load resistance is 5 KQ Caleulate (a) the factor B (b) the input resistance Ry, (© the transconductance and (4) the voltage gain. Sou: =50% MM yp Aly (c) Transconductance, 2mA 20mV (@) The change in output voltage is AV, = RAI, = (5 kQ) (2 mA)=10V AV, = Applied signal voltage = 20. mV Thus, the voltage gain is 10V. 4, = 20 mV Rue =0.1 mho EXAMPLE 34. A transistor is connected in common emitter (CE) configuration. The collector supply is 8 V and the voltage drop across a resistor of 800 Q in the collector circuit is 0.5 V. If the current gain factor (a) is 0.96, find the base current. Sou: The current gain is 0.96 1.33 ‘The collector current is Voltage drop across collector res c Resistance Osv = 3000 x10PA fe But, B= 7, where is base current. 0625x107 A a ctr) =26*10*A=26 pA Exampte 35, A common emitter amplifier has a voltage gain ‘of 50, an input impedance of 200 2 and an output impedance of ‘400 Q. Calculate the power gain of the amplifier. 400 -o( i) ExampLe 36, An n-p-n transistor in a common emitter mode is used as a simple voltage amplifier with a collector connected to load resistance R, and to the base through a resistance R,, ‘The collector-emitter voltage V_.=4 V, the base-emitter voltage Vyg = 0.6 V, current through collector is 4 mA and the current amplification factor B = 100. Calculate the values of R, and R,, 625 x 2= 1250 Power gxin= o(® R, G Rs KH 8Vv Sot: Given, /,=4mA 1.34 Applying Kirchhoft’s second law in loop 1, Voge Bic R = 1x10 O=1kO mA B19 f* tora Applying Kirchhof's second lw in lop cont 8 Vand resistor Ry, “818 ng supply of _ 8-06 i, 4x10 85 kQ EXAMPLE 37. A transistor has a current amplification factor (current gain) of 50. In a CE amplifier circuit, the collector resistance is chosen as 5 kQ and the input resistance is 1k. Calculate the output voltage if input voltage is 0-01. SOL: For transistor amplifier, Y= Er -60/§) (@01)=25V EXAMPLE 38. In a transistor connected in common emitter mode, R, = 4 KO, R,= 1 kO, j,= 1 mA andi voltage gain. Sot: Voltage gain, 4,=B (4) 4) 1x10? \(4 (Ea) ae ExaMpLe 39, For given CE biasing circuit, if voltage aross collector-emitteris 12 V and current gain is 100 and base current is 0.04 mA, then determine the value of collector resistance R. Re Tov X Voc Ven 20-12 Bl, 100x0.04x10" Physics Bryce tacdaus OSCILLATOR Oscillator ‘An oscillator is an electronic device which produces electric i itude, without requitin, oscillation of constant frequency andamp! . any extemal input signal. It converts DC energy obtained from a battery into AC energy in some oscillatory circu Principle of an Oscillator ea i the block diagram of an oscillator. Obviously, ser ony be regarded as the self-sustained transistor amplifier with a postive feedback. Output Feedback cireuit Principle fr an oscillator Essential parts ofa transistor oscillator are () Tank cireuit ‘Atankcircuitis just parallel combination ofan inductance ‘L and a capacitance C. The electric energy once given to it alternately changes between electrostatic energy in the capacitor and the magnetic energy in the inductor. The frequency of electric oscillations in the tank circuit is, 1 . 2nVLC However, the oscillations get damped due to resistive losses inthe inductance, and dielectric losses in the capacitor. ‘Transistor amplifier ‘The oscillations of the tank circuit are fed to the transistor amplifier. The oscillations get amplified due to the amplifying action of the transistor. Feedback circuit ‘To compensate for the energy losses occurring in the tank circuit, the feedback circuit returns (feeds back) a part of the output power of the transistor amplifier to the tank circuit in phase with the input signal. This process is called positive feedback and produces undamped oscillations. ‘The feedback may be done through inductive coupling (mutual inductance), (i) Working of Transistor as an Oscillator(p-n-p) 1. An oscillator is a device which can produce undamped electromagnetic oscillations of desired frequency and amplitude. Semiconductor Electronics: Materials, Devices and Simple Circuits . Tt is a device which delivers A.C. output waveform of desired frequency from DC power even without input signal excitation . Tank circuit containing an inductance Land a capacitance C connected in parallel can oscillate the energy given to it ‘between electrostatic and magnetic energies. However, the oscillations die away since the amplitude decreases rapidly due to inherent electrical resistance inthe circuit. 1. In order to obtain undamped oscillations of constant amplitude, transistor can be used to give regenerative or positive feedback from the output circuit to the input circuit so thatthe circuit losses can be compensated, When key K is closed, collector current begins to grow through the coil L’. Magnetic flux linked with ZL’ as well as L increases as they are inductively coupled. Due to cchange in magnetic flux, induced emf is set up in such a direction that the emitter-base junction is forward biased. ‘This increases the emitter current and hence the collector current. . With the increase in collector current, the magnetic flux across Land L increases. The process continues till the collector current reaches the saturation value. During this, process, the upper plate of the capacitor C gets positively charged, Atthis stage, induced emtfin L becomes zero. The capaci CC starts discharging through the inductor L. The emitter current starts decreasing, resulting in the decrease in collector current. Again, the magnetic flux changes in L’ and J but it induces emf in such a direction that it decreases the forward bias of emitter-base junction. 1. As a result, emitter current further decreases and hence collector current also decreases. This continues till the collector current becomes zero. At this stage, the magnetic flux linked with the coils become zero and hence noinduced emf across L. |. However, the decreasing current after reaching zero value overshoots (goes below zero) and hence the current starts 1.35 increasing but inthe opposite direction. During this period, the lower plate of the capacitor C'gets positively charged, 11. This process continues tll the current reaches the saturation, value in the negative direction. At this stage, the capacitor starts discharging but in the opposite direction (giving positive feedback) and the current reaches zero value from negative value. 12, The cycle again repeats and hence the oscillations are produced. The output is obtained across 1”. 13, ‘The frequency of oscillations is given by f= 1 2aVLC ADVANTAGES AND DISADVANTAGES OF SEMICONDUCTOR DEVICES ‘Advantages ‘© Semiconductor devices are very small in size as ‘compared to the vacuum tubes. Hence, the circuits using semiconductor devices are more compact. ‘© In vacuum tubes, current flows when the filament is heated and starts emitting electrons. So, we have to wait for some time for the operation of the circuit. On the other hand, in semiconductor devices no heating is required and the circuit begins to operate as soon as itis switched on, © Semiconductor devices require low voltage for their ‘operation as compared to the vacuum tubes. So a lot of clectrical power is saved. ‘+ Semiconductor devices do not produce any humming noise which is large in case of vacuum tubes. ‘+ Semiconductor devices have longer life than the vacuum tubes. Vacuum tube gets damaged when its filament is ‘burnt. ‘© Semiconductor devices are shock proof. + The cost of production of semiconductor devices is very small as compared to that of vacuum tubes. * Semiconductor devices can be easily transported as compared to vacuum tubes. Disadvantages * Semiconductor devices are heat sensitive. They get damaged due to overheating and high voltages. So they have to be housed in a controlled temperature toom, # ‘The noise level in semiconductor devices is very high # Semiconductor devices have poor response in high frequency range. ‘Semiconductor Electronics: Materials, Devices and Simple Circuits (3) emitter (4) collector 14, The pat of transistor which is heavily doped to produce a large number of majority carriers, is (1) base (2) emitter @) collector (4) none of these 15, In case of NPN-transistors, the collector current is always less than the emitter current because (1) collector side is reverse biased and emitter forward biased (Q) some electrons are lost in the base and only remaining ‘nes reach the collector @) collector side is forward biased and emitter side is reverse biased (4) collector being reverse biased attracts less electrons 16. When NPN transistor is used as an amplifier, (1) electrons move from base to collector (2) holes move from emitter to base G) electrons move from collector to base (4) holes move from base to emitter 17. In a common emitter transistor, the current gain is 80. ‘What is the change in collector current, when the change in base current is 250 WA? (1) 80x 250 pA Q) 250-80) wa G) (250+ 80) pA (4) 250/80 yA 18, In an NPN transistor, the collector current is 24 mA. If 80% of electrons reaches collector, its base current (in mA) is 1) 36 2 26 @) 16 6 19. The phase difference between input and output voltages of CE amplifier is Me 2 9° 3) 180° (4) 270 20. The transfer ratio ofa transistor is 50. The input resistance of the transistor when used in the common emitter is configuration is 1 k©. The peak value for an AC input voltage of 0.01 V peak is (1) 100 pa, (2) 0.01 ma (3) 0.25 mA (4) 500 pa. 21, In the CB mode of a transistor, when the collector voltage is changed by 0.5 V, the collector current changes by 0.05 mA. The output resistance will be () 10kQ (2) 20k @) SkQ (4) 2.542 137 Pete =PUi Oa eed The term digital is derived from the way in which computers perform operations using digits. Initially, applications of digital electronics were confined to computer systems. Now a days, digital techniques are applied in many areas, such as telephony, radar, medical instruments, navigation and military systems etc. Digital electronics involves circuits and systems in which there are only two possible states which are represented by voltage levels. Other circuit conditions such as current levels, open or closed switches can also represent the two states, ‘Analog Signal The signal current or voltage is in the form of continuous, time ‘varying voltage or current (sinusoidal). Such signals are called continuous or analog signals. A typical analog signal is shown in figure below. 4 v Digital Signal and Logic Levels ‘A digital signal (pulse) is shown in figure below. sv - ov It has two discrete levels, ‘High’ and “Low”. In most cases, ‘the more positive of the two levels is called HIGH and is also referred to as logic 1. The other level becomes low and is also called logic 0. This method of using more positive voltage level as logic 1 is called a positive logic system. A voltage SV refers to logic | and 0 V refers to logic 0. Digital Circuit ‘An electrical or electronic circuit which operates only in two states (binary mode) namely ON and OFF is called a Digital Circuit. In digital system, high value of voltage such as +10 V or +5 V is represented by ON state or I (state) whereas low value of voltage such as 0 V or ~SV or -10 V is represented by OFF state or 0 (state). Boolean Algebra George Boole developed an algebra called Boolean Algebra to solve logical problems, In this, 3 logical operations viz. 1.38 OR, AND and NOT are performed on the variables. The two values or states represent either ‘TRUE’ or ‘FALSE; ‘ON’ or ‘OFF";"HIGH’ or *LOW'; ‘CLOSED’ o ‘OPEN’; 1 or 0 respectively. GIC GATES 1. Gate is a digital circuit with one or more inputs but wi only one output. The output appears only for certain ‘combination of input logic levels. In other words, output cof a logic gate has certain logical relation with inputs. 2. The numbers 0 and I represent the two possible states of a logic circuit. The two states can also be refered to as ‘ON and OFF" or ‘HIGH and LOW’ or ‘TRUE and FALSE’. Operation of given gate is expressed in terms of Boolean Expression. 3. The logic gates are diodes and transistors. 4, Each logic gate is represented by its characteristic symbol. ‘5. The operation ofa logic gate is indicated in a table, known astruth table, This table contains all possible combinations of inputs and the corresponding outputs. 6. A logic gate is also represented by a boolean algebraic expression. Boolean algebra is a method of writing logical equations showing how an output depends upon the combination of inputs. Boolean algebra was invented by using semiconductor p-n junc George Boole. 7. There are three basic logic gates. They are: (1) OR gate Q) AND gate (3) NOT gate OR Gate the state I ifone or more 1, The output of an OR gate a inputs attain the state 1. 2. Logie symbol of OR gate: 3. The boolen expression of OR gate is Y=A+B This is read as Y equals A OR B. 4, Truth table of a two input OR gate: Er=tere|= =e ls [’] 0 1 1 1 Physics 5, Electronle cireuit of OR gate ; i in figure below, the positive voltage (+5 V) + onde ohighinpul 1 (State) and the negate terminal of the battery is grounded and corresponds to ow input, ie., 0 (state). ii. In figure below, dotted boundary encloses OR gate, OR gate is made up of two diodes. ili, Working: Case |: Both A and B are given 0 input and the diodes do not conduct current. Hence, no output is across R,, Y=0. Case 2: Ais given 0 and B is given 1. Diode D, does not conduct current (cut-off) but D, conducts. Hence, ‘output (5 V) is available across R,, ' Case3: Ais given | and Bis given 0. Diode D, conducts current but D, does not conduct. Hence, output (5 V) is available across R, i.e. Y= 1. Case 4: A and B are given 1. Both the diodes conduct current. However, output (only 5 V) is available across Ry ie, ¥=1 AND Gate 1. The output of an AND gate attains the state | if and only if all the inputs are in state 1. 2. Logic symbol of AND gate: A_+ B 3. The boolean expression of AND gate is Y=AB Itis read as ¥ equals A AND B. 4, Truth table of a two-input AND gate: 4]8]¥ Feo 5. Electronic cireult of AND gate In figure below, dotted boundary encloses AND gate AND gate also consists of two diodes connected in manner different than that in OR gate. Unlike OR gate, AND gate requires a dedicated power supply. Att \ D, Case 1: Both A and B are given 0 input and the diodes ‘conduct current (Forward biased). Since the current is drained to the earth, hence, no output across R, i.e. Y= 0 Case 2: is given 0 and B is given 1. Diode D, being forward biased conducts current but D, does not conduct. However, the current from the output battery is drained through D,. So, Y= 0. Case 3: 4 is given 1 and B is given 0. Diode D, does not conduct current but D, being forward biased conducts However, the current from the output battery is drained through D,. Hence, no output is available across R,. i. ¥=0. Case 4: A and B are given 1. Both the diodes do not conduct, current. The current from the output battery is available across R, and output circuit. Hence, there is voltage drop (5 V) across R,. i. Y= 1. NOT Gate 1, The output of a NOT gate attains the state 1 iff and only if the input does not attain the state 1 2. Logic symbol of NOT gate: 4 y 3. The boolean expression is yea It is read as ¥ equals NOT A. 4. Truth table of NOT gate: A|¥ oft tho NOT gate has only one input. ‘Semiconductor Electronics: Materials, Devices and Simple Circuits 1.39 5. Electronic clreuit of NOT gate 1m figure below, dotted boundary encloses NOT gate, It consists of transistor. NPN transistor is connected to biasing batteries, through base resistor (R,) and collector resistor (R,). Emitter is ditectly earthed. Input is given through the base and the output is tapped across the collector. Case 1: Ais given 0 input. Inthe absence of forward bias to the P-type base and N-type emitter, the transistor is in cut- off mode (does not conduct current). Hence, the current from the collector battery is available across the output unit. Therefore, voltage drop of 5 V is available across Y, ie Case 2: 4 is given 1 input by connecting the positive terminal ofthe input battery. P-type base being forward biased makes the transistor in conduction mode. The current supplied by the collector battery is drained through the transistor to the earth. Therefore, no output is available across Y, ie. Y=0 Combination of Gates ‘The three basic gates (OR, AND and NOT) when connected in various combinations give us logic gates such as NAND, NOR ‘gates, which are the universal building blocks of digital circuits. NAND Gate |. Logic symbol of NAND gate: 4 Input B 2. The boolean expression of NAND gate is y= 4B 3. Truth table of a NAND gate: BLY He os 0 l 0 1 1.40 NOR Gate 1, Logic symbol of NOR gate: 2, The boolean expression of NOR gate is +B 3. Truth table of a NOR gate: a 0 0 1 L Soc Ferofe EAC 1ieaN Basic OR, AND, and NOT operations are given below: oR A+O=A Ati=l AtA=A Boolean algebra obeys commutative, associ laws as given below: A.AzA tiveand distributive © Commutative laws: A+B=B+A AB=BA Associative laws: A+ (B+ O=(A+B)+C A.(B.0)=(A.B).C Distributive laws: ABH O)=AB+AL ‘© Some other useful identities: () A+ AB= (i) AA+B)=A (iii) A+(AB)= A+B (iv) A(A+B)=AB Physics — (vy) A+B.0= (4+ B.AFO C+BAtBC (vi) (At BMA+C)=A ‘© De Morgan's theorem: First theorem: Second theorem: A.B ic 2 calls ‘The NAND or NOR gate is the digital circuits. Repeated use of ‘Therefore, any digi entirely from NAND ot NOR gates. We repeated use of NAND (and NOR) gates Wi ‘gates. block of all 'NAND gates (or NOR gates) jtal system can be achieved shall show how the il gives us different NOT gate from a AND gate ‘When all the inputs of a NAND gate ‘are connected together, as shown in the figure, we obtain a NOT gate. ¥ ‘AND gate from NAND gates If NAND gate is followed by a NOT gate (-c., @ single input AND gate), the resulting circuit is AND gate as shown in figure and truth table given shows how an AND gate has been obiained from NAND gates. ee s ‘Truth table of a single input NAND gate: Sarre ¥ 1 jt 1 0 -eoox ‘Semiconductor Electronics: Materials, Devices and Simple Circuts (OR gate from NAND gates If we invert the inputs A and B and then apply them to the [NAND gate, the resulting circuit is an OR gate a4 <4 Sere F vse A\B\A B ofojtfiy a Jolijilo| o 1jojoli| o 1ifolol o NOT gate from NOR gate ‘When all the inputs of a NOR gate are connected together as shown in the figure, we obtain a NOT gate. D> AND gate from NOR gates Ifwe invert the inputs 4 and B and then apply them to the NOR gate, the resulting circuit is an AND gate. 4 A Y=A+B=AB S B A|B\A|B| A+B | A+B O;o;lit 1 0 O}1j1io0 1 0 1\o;o}1 1 0 1ji jojo} 0 1 OR gate from NOR gate If a NOR gate is followed by a single input NOR gate (NOT gate), the resulting circuit is an OR gate. 1M Exclusive-OR gate (XOR gate) 1. The output of a two-input XOR gate attains the state 1 if ‘one and only one input attains the state 1 2. Logie symbol of XOR gate: —_) > 3. The Boolean expression of XOR gate is, Y= AB+AB =A®@B 4, Truth table of an XOR gate z Exclusive - NOR gate (XNOR gate) 1. The output is in state 1 when its both inputs are same, that is, both 0 or both 1. 2. Logie symbol of XNOR gate: 3. ‘The boolean expression of XNOR gate is Y=AB+AB ot A®B or AOB 4, Truth table of a XNOR gate: BUY 1 o| eS ols = 0 1 0 ‘Summary of Logic Gates NAMES: ‘SYMBOL ‘BOOLEAN TRUTH ELECTRICAL ANALOGUE CIRCUIT DIAGRAM (PRACTICAL EXPRESSION ‘TABLE REALISATION) a[a7]| —4 h— , ; aT . on SSH] reeee [fol] PL , Jk | |e Vjolt a dita - atotol | 47 ey. 4 lax PL | rae |folito kh} rene , |] o I 7 aa p+ eA 0 Ltr} * 72 : |worcor+ | 4 ojo So of: lo 5 NOT) F 1folo A illo | 7 EF a nr opoyt 4 wo | Sop | offi Gy NoT) a rfofif} Le lilo 7a] aoe ‘ apoyo (Exclusive ~~ y=4p+4B |[olt]1 oR) rola lilo A\BIY XNOR 4 y ¥=AB+AB |/9/0}1 texave | J) ot |“ Wola NOR) B Y=A@B 1jo}o bh ‘Semiconductor Electronics: Materials, Devices and Simple Circuits EXAMPLE 40. Sketch the output ¥ from a NAND gate having inputs A and B given below: Sou: i Forr1,=0, B= 1; Hence ¥ x a A (input) B ii (input) (oupur) seston | ee ‘An integrated circuit (IC) is a small semiconductor wafer on which thousands to millions of tiny resistors, capacitor, and transistors are fabricated. ICs are produced on a piece of semiconductor crystal (or chip) by a process called photolithography. The IC was first invented by Jack Kilky at ‘Texas Instruments in 1958 and he was awarded Nobel Prize for this in 2000. ‘An IC can function as an amplifier, oscillator, timer, counter, computer memory, oF microprocessor. ICs are found in almost all electronic devices like personal computers, televisions, cell phones, music players to name a few. In addition to it, electromechanical devices, like automobiles and industrial ‘machinery contain ICs to automate several tasks. 143 Many of devices use a special kind of IC called microprocessor. ‘A microprocessor is an integrated circuit (IC) which incorporates core functions of a computer's central processing unit (CPU). It is a programmable multipurpose silicon chip, clock driven, register based, accepts binary data as input and provides output after processing it as per the instructions stored in the memory. ‘The integration of huge number of transistors into a tiny chip results in citcuits that are faster, smaller, efficient and less expensive than those made of individual electronic components. ‘The miniaturisation that made the modern personal computer possible could never have happened without the ICs. Over the years, the complexity of ICs has increased while its size has ‘decreased. In the past five decades, a dramatic miniaturisation in computer technology has made modern day computers faster and smaller. The number of transistors per chip has risen exponentially and each year computers are becoming, more powerful, yetcheaper than the year before. Theentire Information ‘Technology (IT) industry hinges on semiconductors. In the 1970s, Gordon Moore, co-founder of INTEL, pointed out that the memory capacity of a chip (IC) approximately doubled every one and a half years. This is popularly known as Moore's law. The explosive growth in the semiconductor industry and ‘computer technology is best expressed by a famous quote from Gordon Moore: “Ifthe auto industry advanced as rapidly as the semiconductor industry, a Rolls Royce would get half a million miles per gallon, and it would be cheaper to throw it away than 10 park it” CONCEPT APPLICATION 4 1, The circuit shown in figure below will act as 4 B (1) OR gate 2) AND gate (3) XOR gate (4) none of the above 2, In boolean algebra, A + B= ¥ implies that (1) sum of 4 and Bis ¥ (2) Yexists when d or B exists or when both 4 and B exist (3) Yexists only when 4 and B both exist (4) Yexists when 4 or B exists but not when both 4 and B exist 3. In Boolean algebra, which of the following is wrong? (140-1 Q) o+1=1 @)l+t=t (4) None of above 4. The output of a 2 input OR gate is given to a NOT gate. (1) OR gate (3) NOR gate (2) NOT gate (4) XOR gate

You might also like